Difference between revisions of "2020 AMC 10B Problems/Problem 13"

(Created page with "==Problem== Andy the Ant lives on a coordinate plane and is currently at <math>(-20, 20)</math> facing east (that is, in the positive <math>x</math>-direction). Andy moves <ma...")
 
(Problem)
Line 1: Line 1:
 
==Problem==
 
==Problem==
 
Andy the Ant lives on a coordinate plane and is currently at <math>(-20, 20)</math> facing east (that is, in the positive <math>x</math>-direction). Andy moves <math>1</math> unit and then turns <math>90^{\circ}</math> degrees left. From there, Andy moves <math>2</math> units (north) and then turns <math>90^{\circ}</math> degrees left. He then moves <math>3</math> units (west) and again turns <math>90^{\circ}</math> degrees left. Andy continues his progress, increasing his distance each time by <math>1</math> unit and always turning left. What is the location of the point at which Andy makes the <math>2020</math>th left turn?
 
Andy the Ant lives on a coordinate plane and is currently at <math>(-20, 20)</math> facing east (that is, in the positive <math>x</math>-direction). Andy moves <math>1</math> unit and then turns <math>90^{\circ}</math> degrees left. From there, Andy moves <math>2</math> units (north) and then turns <math>90^{\circ}</math> degrees left. He then moves <math>3</math> units (west) and again turns <math>90^{\circ}</math> degrees left. Andy continues his progress, increasing his distance each time by <math>1</math> unit and always turning left. What is the location of the point at which Andy makes the <math>2020</math>th left turn?
 
 
<math>\textbf{(A)}\ (-1030, -994)\qquad\textbf{(B)}\ (-1030, -990)\qquad\textbf{(C)}\ (-1026, -994)\qquad\textbf{(D)}\ (-1026, -990)\qquad\textbf{(E)}\ (-1022, -994)</math>
 
<math>\textbf{(A)}\ (-1030, -994)\qquad\textbf{(B)}\ (-1030, -990)\qquad\textbf{(C)}\ (-1026, -994)\qquad\textbf{(D)}\ (-1026, -990)\qquad\textbf{(E)}\ (-1022, -994)</math>
 
  
 
==Solution==
 
==Solution==

Revision as of 18:45, 7 February 2020

Problem

Andy the Ant lives on a coordinate plane and is currently at $(-20, 20)$ facing east (that is, in the positive $x$-direction). Andy moves $1$ unit and then turns $90^{\circ}$ degrees left. From there, Andy moves $2$ units (north) and then turns $90^{\circ}$ degrees left. He then moves $3$ units (west) and again turns $90^{\circ}$ degrees left. Andy continues his progress, increasing his distance each time by $1$ unit and always turning left. What is the location of the point at which Andy makes the $2020$th left turn? $\textbf{(A)}\ (-1030, -994)\qquad\textbf{(B)}\ (-1030, -990)\qquad\textbf{(C)}\ (-1026, -994)\qquad\textbf{(D)}\ (-1026, -990)\qquad\textbf{(E)}\ (-1022, -994)$

Solution

Video Solution

See Also

2020 AMC 10B (ProblemsAnswer KeyResources)
Preceded by
Problem 12
Followed by
Problem 14
1 2 3 4 5 6 7 8 9 10 11 12 13 14 15 16 17 18 19 20 21 22 23 24 25
All AMC 10 Problems and Solutions

The problems on this page are copyrighted by the Mathematical Association of America's American Mathematics Competitions. AMC logo.png